Calculus Review - Tangent of function and find y-intercept

Click For Summary
The discussion focuses on finding the y-intercept of a tangent line to the function g(ξ) = (4/π) arctan(ξ) at the point ξ0 = 1.30. The slope of the tangent line is calculated as 0.473323 by taking the derivative and evaluating it at ξ = 1.3. The y-value at this point is determined to be 1.16514. However, the calculation for the y-intercept using the equation y = mx + b is incorrect, as the resulting value for b is stated to be 1.89, which contradicts the expected outcome. The discussion concludes with a reminder to double-check calculations for accuracy.
oddjobmj
Messages
305
Reaction score
0

Homework Statement


Consider g(ξ) = [(2H)/π] arctan(ξ).
Plot a graph of the function g(ξ).
Imagine a line that passes through the point on the curve at ξ0 = 1.30, and which is tangent to the curve at that point. Where does the tangent line intersect the vertical axis?
[DATA: H = 2.00 ; ξ0 = 1.30 .]

Homework Equations


y=mx+b

The Attempt at a Solution


Firstly I know that H=2 so I can easily simplify the question to (4/π)*arctan(ξ).

I then want to find the slope so I take the derivative of that function and get 4/(πξ^2+π) then plug in ξ=1.3 to find the slope at that point. The result is: 0.473323

I can also plug ξ=1.3 into the original equation to find the y value at that point. The result is: 1.16514

Plugging in those values to y=mx+b I get 1.16514=(.473323)(1.3)+b. The resulting y-intercept (b) is 1.89.

Unfortunately, this is not correct! I am obviously making a silly mistake somewhere. Is anyone able to point out what I did wrong?

Thank you!
 
Physics news on Phys.org
You didn't evaluate y = mx + b correctly.

If y = 1.16514 and mx = 0.473323*1.3, then b must be less than 1.16514
 
I really have no idea what to say. I'll start with a thank you. Sometimes the most obvious things...
 
You're welcome. Good luck with the rest of your review.
 
Question: A clock's minute hand has length 4 and its hour hand has length 3. What is the distance between the tips at the moment when it is increasing most rapidly?(Putnam Exam Question) Answer: Making assumption that both the hands moves at constant angular velocities, the answer is ## \sqrt{7} .## But don't you think this assumption is somewhat doubtful and wrong?

Similar threads

  • · Replies 4 ·
Replies
4
Views
1K
Replies
3
Views
2K
Replies
13
Views
4K
  • · Replies 10 ·
Replies
10
Views
3K
  • · Replies 11 ·
Replies
11
Views
5K
  • · Replies 1 ·
Replies
1
Views
1K
  • · Replies 9 ·
Replies
9
Views
2K
  • · Replies 8 ·
Replies
8
Views
2K
  • · Replies 4 ·
Replies
4
Views
2K
Replies
7
Views
3K